Lipschitz Condition: Does $f(t,y)$ Satisfy? Find Constant

In summary, the function "$f(t,y)$" does not satisfy the Lipschitz condition uniformly for $y$. The Lipschitz constant is $1$.
  • #1
evinda
Gold Member
MHB
3,836
0
Hello! (Wave)

Does the following $f(t,y)$ satisfy the Lipschitz condition as for $y$, uniformly as for $t$? If so, find the Lipschitz constant.

$$f(t,y)=\frac{|y|}{t}, t \in [-1,1]$$

I have tried the following:

$$\frac{|f(t,y_1)-f(t,y_2)|}{|y_1-y_2|}=\frac{|y_1|-|y_2|}{t|y_1-y_2|} \leq - \frac{|y_1|-|y_2|}{|y_1-y_2|}= \frac{|y_2|-|y_1|}{|y_1-y_2|} \leq \frac{|y_1-y_2|}{|y_1-y_2|}=1$$

Thus $f$ satisfies the Lipschitz condition and the Lipschitz constant is equal to $1$.

Is it right? (Thinking)
 
Physics news on Phys.org
  • #2
evinda said:
$$\frac{|f(t,y_1)-f(t,y_2)|}{|y_1-y_2|}=\frac{|y_1|-|y_2|}{t|y_1-y_2|} \leq - \frac{|y_1|-|y_2|}{|y_1-y_2|}$$
These equality and inequality are incorrect.
 
  • #3
Evgeny.Makarov said:
These equality and inequality are incorrect.

Is it right now?

$\frac{|f(t,y_1)-f(t,y_2)|}{|y_1-y_2|}= \frac{|\frac{|y_1|}{t}-\frac{ |y_2|}{t} |}{|y_1-y_2|}= \frac{||y_1|-|y_2||}{|t| |y_1-y_2|} \leq \frac{|y_1-y_2|}{|t| |y_1-y_2|}=\frac{1}{|t|} \overset{t \to 0}{\to } +\infty$

So we deduce that $f(t,y)$ doesn't satisfy the Lipschitz condition.
 
  • #4
The formulas are correct, but the conclusion is incorrect without the word "uniformly".
 
  • #5
Evgeny.Makarov said:
The formulas are correct, but the conclusion is incorrect without the word "uniformly".

So do we want to prove that the $\frac{|f(t,y_1)-f(t,y_2)|}{|y_1-y_2|}$ converges uniformly to $+\infty$, i.e. for any $t$ we pick?
But doesn't it only happen for $t \to 0$? Or am I wrong?

Or did you mean with uniformly that the limit is $+\infty$ for any $y_1, y_2 \in \mathbb{R}$ and for $t \to 0$?
 
Last edited:
  • #6
This problem does not require talking about convergence, especially uniform convergence. It is about uniform Lipschitz property. For each particular $t\ne0$ the function $f(t,y)$ is Lipschitz.
 
  • #7
Evgeny.Makarov said:
This problem does not require talking about convergence, especially uniform convergence. It is about uniform Lipschitz property. For each particular $t\ne0$ the function $f(t,y)$ is Lipschitz.

Ah I see... Thanks a lot! (Smile)
 

Related to Lipschitz Condition: Does $f(t,y)$ Satisfy? Find Constant

1. What is the Lipschitz condition?

The Lipschitz condition is a mathematical concept that ensures the existence and uniqueness of solutions to differential equations. It states that the rate of change of a function cannot vary too quickly, and is bounded by a constant value.

2. How is the Lipschitz constant calculated?

The Lipschitz constant is calculated by finding the maximum absolute value of the derivative of the function with respect to its independent variable. This value is denoted by the symbol L and is used to determine if the Lipschitz condition is satisfied.

3. What does it mean if a function satisfies the Lipschitz condition?

If a function satisfies the Lipschitz condition, it means that it meets the criteria for the Lipschitz condition and therefore has a unique solution. This ensures that the solution to the differential equation is stable and will not change drastically with small changes in initial conditions.

4. What happens if a function does not satisfy the Lipschitz condition?

If a function does not satisfy the Lipschitz condition, it may not have a unique solution to the differential equation. This can lead to instability in the solution and make it difficult to predict the behavior of the system.

5. Is the Lipschitz condition always satisfied?

No, the Lipschitz condition is not always satisfied. It depends on the specific function and its derivatives. Some functions may have a Lipschitz constant that is too large, making it difficult to determine a unique solution, while others may not have a Lipschitz constant at all.

Similar threads

Replies
3
Views
2K
Replies
18
Views
2K
  • Calculus and Beyond Homework Help
Replies
2
Views
394
  • Differential Equations
Replies
2
Views
1K
  • Introductory Physics Homework Help
Replies
8
Views
768
Replies
12
Views
1K
  • Calculus
Replies
12
Views
2K
Replies
3
Views
956
  • Calculus
Replies
2
Views
1K
Replies
1
Views
1K
Back
Top